Please someone help me

Please Someone Help Me

Answers

Answer 1

Answer:

92.4 m^2

Step-by-step explanation:

Area of Trapezoid: h(b1+b2)/2

6.6 (14.2 + 7.6 + 6.2) / 2

= 6.6 (28) / 2

= 184.8/2

= 92.4 m^2

Answer 2
If I’m right it equals 25.08m sq. Hope this helps!!

Related Questions

Jake is earning money for a new TV. So far, he has put $54.29, $87.20, and $21.44 into his savings account. How much money has Jake saved?

Answers

add all the values together

54.29+87.20=141.49

141.49+21.44=162.93

Answer:

$162.93

Step-by-step explanation:

Do 54.29 + 87.20 + 21.44 and get 162.93 than put this $ in front of you answer  then you have $162.93

I need help with is question ASAP please

Answers

The y-intercept looks like it's 0 and the slop looks like 6/2 or 3 or maybe 7/2

Tiffany cells 2 kinds of homemade tomato sauce

Answers

Answer:

b

Step-by-step explanation:

Let x represent the number of quarts of Tuscan sauce and

y represents the number of quarts of marinara sauce Tiffany makes.

A quart of Tuscan sauce requires 6 tomatoes and 1 cup of oil

x quarts requires 6x tomatoes and 1x cups of oil

A quart of her marinara sauce requires 5 tomatoes and 1.25 cups of oil

y quarts requires 5y tomatoes and 1.25 y cups of oil

She has 45 tomatoes and 10 cups of oil on hand.

So the constraints are

6x+5y≤45

1x+1.25y≤10

x>=0 and y>=0

After drawing a diagram of what's required to make 1 quart of Tuscan and 1 quart of Marinara sauce, you can more clearly choose answer choice (B) 6x + 5y < (or equal to) 45, x + 5/4y < (or equal to) 10, x>(or equal to)0, y>(or equal to)0.

please mark brainliest :)

the answer is B !!! hope it’s correct

Which graph represents a function with direct variation?
help asap!!

Answers

The right answer is the second one

a) the quadratic x^2-4x-21 can be written in the form (x+a)^2+b find the values of and and b

b) hence, or otherwise, solve the equation x^2-4x-21=0

Answers

Part a. A=-2. B=-25
Part b solutions; x=7 x=-3

Find the value of x in the triangle pair below

Answers

Answer:

x=16.97056274... or 12[tex]\sqrt{2}[/tex]

Step-by-step explanation:

the hypotenuse of a 45, 45, 90 triangle is the length of the leg times the square root of 2

BEST ANSWER GETS TO CHOOSE BRAINLESST OR FOLLOW!​

Answers

Answer:13 is>

14 is =

15 is >

16 is<

17 is<

18 is =

Step-by-step explanation:

I just know plz mark brainliest



A new car is purchased for 23400 dollars. The value of the car depreciates at 11.5% per year. To the nearest year, how long will it be until the value of the car is 12700 dollars?

Answers

Answer: 5 years

Step-by-step explanation:

Using same formula as for Compound Interest:

12700 = 23400 (1-.115)^t

12700 = 23400 x 0.885^t

0.885^t = 127/234

Convert decimal into fraction:

177/200^t = 127/234

Take logarithm of both sides of thr equation:

t = log 177/200 (127/234)

t = 5.00242

If g(x) = -4x + 5, find g(2x-1).

Answers

Answer:

- 8x + 9

Step-by-step explanation:

To evaluate g(2x - 1) substitute x = 2x - 1 into g(x), that is

g(2x - 1)

= - 4(2x - 1) + 5 ← distribute parenthesis and simplify

= - 8x + 4 + 5

= - 8x + 9

Answer:

= - 8x + 9

Step-by-step explanation:

= - 8x + 4 + 5

= - 8x + 9

Is the relation a function?
{(-6, -1), (5,-1), (0, -1), (-2, -1), (3, -1)}

Answers

Answer:

yes it is a function

Step-by-step explanation:

For that no two x values are the same with different y values

What is the midpoint of the segment below? (2,3)(-3,-2)

Answers

Answer:

(-0.5, 0.5)

Step-by-step explanation:

If those two points are endpoints, just use the midpoint formula.

(-0.5, 0.5)

Answer:

(-1/2,1/2)

Step-by-step explanation:

To find the midpoint, add the x coordinates and divide by 2

(2+-3)/2 = -1/2

add the y coordinates and divide by 2

(3+-2)/2 = 1/2

The midpoint is (-1/2,1/2)

PLEASE HELP WILL GIVE BRAINLIEST AND 20 POINTS

solve this system of equations

3y-5x=12
y=1/3x

Answers

Answer:

x=-3

y=-1

(-3,-1)

Answer:

(-3, -1).

Step-by-step explanation:

I have attached the work to your problem.

Please see the attachment below.

I hope this helps!

Find the surface area of the cylinder.
7 cm
5 cm

Answers

Answer:

A≈527.79

Step-by-step explanation:

A=2πrh+2πr2=2·π·7·5+2·π·72≈527.78757

A few years ago, a country consumed 19,400,000 barrels of oil per day. That same year, there were about 119,000,000 households in that country. A barrel of oil is 42 U.S. gallons. Estimate how many gallons each household in the country consumed each day that year by rounding down all three numbers. (Round 19,400,000 barrels to 19,000,000 barrels, 119,000,000 households to 115,000,000 households, and 42 gallons to 40 gallons. Round your answer to two decimal places.)

_________ gal/household/day

Calculate the answer without using estimation and compare the two results. (Round your answer to two decimal places.)
___________gal/household/day

Answers

Following are the calculation to fill in the blank:

For Part a)

Each home in this town consumed

[tex]\to 40 \times \frac{19000000}{115000000}\\\\\to 40 \times \frac{19}{115}\\\\\to 40 \times 0.165\\\\\to 6.6 \ \text{gallons of oil per day}[/tex]

For Part b)

Without completing the sentence,

[tex]\text{Total consumption = 19200000 barrels}\\\\\text{Total number of households = 118000000}[/tex]

As a result, each household consumed.

[tex]\to \frac{19200000}{118000000} \\\\\to \frac{192}{1180} \ (barrels)\\\\[/tex]

In the given scenario, each barrel includes 42 gallons of oil. Therefore, every household consumed [tex]42 \times \frac{192}{1180} = 6.83 \ gallons[/tex] oil each day.

Learn more:

brainly.com/question/14798687

Hence, each household will consume 6.60 gallons per day.

Total Households of that country =115000000 (approx.)

Total barrels used per day =19000000 (approx.)

Given 1 barrel = 40 gallons (approx.)

So total gallons used per day = 19000000 x 40 =760000000

So each household will consume gallons per day = total gallons used per day / Total household of that country

=[tex]=\frac{760000000}{ 11500000}[/tex]

=6.60 gallons/household/day

So each household will consume 6.60 gallons per day.

Learn More: https://brainly.com/question/12272570

Micheal has 1/2 of a yard of fabric to make book covers. Each book cover is made from 1/8 of a yard of fabric. How many book covers can Micheal make?

Answers

Answer:

4

Step-by-step explanation:

[tex]\frac{1}{2}[/tex] ÷ [tex]\frac{1}{8}[/tex] = 4

Marcia has two credit cards and would like to consolidate the two balances into one balance on the card with the lower interest rate. The table below shows the information about the two credit cards Marcia currently uses.


Card A
Card B
Amount
$1,389.47
$1,065.32
APR
16%
12%
Monthly Payment
$39.38
$28.05

After 4 years, approximately how much will Marcia have saved in interest by consolidating the two balances?
a.
$1,890.24
b.
$133.92
c.
$543.84
d.
$1,346.40

ANSWER IS B

Answers

Answer:

Option B ,$133.92  is correct

Step-by-step explanation:

By consolidating both balances in Card B,APR of 12% would apply to funds in card A as well

Let us approach the question by solving for monthly payment of  Card A using   12% APR as follows while making use of excel pmt formula:

=pmt(rate,nper,-pv,fv)

rate is the monthly rate on the card which is 12%/12=1%

nper is the number of months of payment which is 12 months * 4 years=48

pv is the current balance in the card which is $1,389.47

fv is the total amount of monthly repayment which is taken as zero

=pmt(1%,48,-1389.47,0)=$ 36.59  

Using 16% monthly payment is $39.38  which is given already

Total monthly payments for 16%=$39.38*48=$1890.24

Total monthly payments for 12% APR=$36.59*48=$1756.32

savings in total repayments=savings in interest=$1890.24 -$1756.32 =$133.92

Answer:

B. $133.92

Step-by-step explanation:

Just took the test and got it right

Line I and h intersect at what point

Answers

Answer:

please show where the lines are graphed in a picture

Step-by-step explanation:

Please Help me it’s geometry

Answers

Answer:

Option C.

Step-by-step explanation:

Given: In [tex]\Delta OPQ,m\angle O=107^{\circ},m\angle P=28^{\circ} [/tex].

In [tex]\Delta OPQ[/tex],

[tex]m\angle O+m\angle P+m\angle Q=180^{\circ}[/tex]    (Angle sum property)

[tex]107^{\circ}+28^{\circ}+m\angle Q=180^{\circ}[/tex]

[tex]135^{\circ}+m\angle Q=180^{\circ}[/tex]

[tex]m\angle Q=180^{\circ}-135^{\circ}[/tex]

[tex]m\angle Q=45^{\circ}[/tex]

Now,

[tex]m\angle O>m\angle Q>m\angle P[/tex]

In a triangle, the greatest angle has largest opposite side and smallest angle has smallest opposite side. So, we conclude that

[tex]PQ>OP>QO[/tex]

Therefore, the correct option is C.

00:00
Brice is finding the sum of 468 and 241 by breaking it into smaller problems
He uses place value and finds the sums of the hundreds, tens, and ones.
What is the sum of the tens? Enter your answer in the box.
1​

Answers

468+241=709 so 0 is in the tenth place because 6+4= 10 and one gets carried over to the 4+2

PLLZZZZZZZZZZZZ HELP ME :(((((((((

Answers

Answer:

Least children=9

Step-by-step explanation:

there can be at least 1 row of 6 kids with 3 left so that is 9 kids. and there can be at least 1 row of 8 kids with 3 left so there is 11 kids. So the least number of kids possible is 9.

Becky made 4 quarts of chicken noodle soup in a big pot, then served 112-cup bowls of soup to 6 people. How much soup is left in the pot?

Answers

Answer:

1qt 3c

Step-by-step explanation:

HELP ASAP it says my questions should be at least 20 characters long so what I’m saying right now doesn’t mean anything.

Answers

Answer: See pic above for answer. I got it from Photomath

Plz put brainliest

please help as soon as possible
I WILL MARK YOU AS BRAINLIEST ​

Answers

Answer:

Last choice

Step-by-step explanation: Subtract 11 and then divide by 3. A would be greater than -2 and last choice shows that

What is the equation in slope-intercept form of a line with slope of 5 and y-intercept of 3?

Answers

y=5x+3

Because the starting value(y intercept is 3) and the slope(x) is 5


[tex]( {2}^{ - 1} + {3}^{ - 1} )^{2} [/tex]
solve.

will give the brainliest​

Answers

Answer:

25/36

Step-by-step explanation:

(2^-1 + 3^-1)^2

(1/2 + 1/3)^2

(5/6)^2 = 25/36

25/36 or .694

2^-1= 1/2
3^-1=1/3

Add the together
5/6 square it and you get 25/36

If f(x) = 2x + 1 and g(x) = x-2 what is the value of f(g(f(3)))? This is so confusing plz help me will mark brainliest A) 1 B) 3 C) 5 D) 7 E) 11

Answers

Answer:

11

Step-by-step explanation:

f(3) = 2*3+1 = 6+1 = 7

Then find g(7)

g(7) = 7-2 = 5

Then find f(5)

f(5) = 2*5 +1 = 10+1 = 11

f(g(f(3))) = 11

Answer:

11

Step-by-step explanation:

f(x)=2x + 1

f(3)= 2.3 + 1

f(3)=7

f(g(f(3))) = f(g(7))

Now, we have to find g(7)

g(x)=x -2

g(7)= 7 -2

g(7)=5

f(g(f(3))) = f(g(7)) = f(5)

now we have to find f(5)

f(x)=2x + 1

f(5)=2.5 + 1

f(5)=11

Hope this helps ^-^

which of the following represents 8 square root x^3 in exponential form

Answers

Answer:

x^3/8

Step-by-step explanation:

Use  n √ a^ x  = a ^x/ n  to rewrite  8 √ x ^3  as  x ^3/ 8 .

Find the solution(s) to x^2- 14x + 49 = 0.
O A. x=-2 and x = 7
B. x= -1 and x = 14
C. x= 7 only
D. x = 7 and x = -7

Answers

Answer:

[tex] \boxed{C. \: x = 7 \: only} [/tex]

Step-by-step explanation:

[tex] = > {x}^{2} - 14x + 49 = 0 \\ \\ = > {x}^{2} - (7 + 7)x + 49 = 0 \\ \\ = > {x}^{2} - 7x - 7x + 49 = 0 \\ \\ = > x(x - 7) - 7(x - 7) = 0 \\ \\ = > (x - 7)(x - 7) = 0 \\ \\ = > {(x - 7)}^{2} = 0 \\ \\ = > x - 7 = 0 \\ \\ = > x = 7[/tex]

Pls help me on this................

Answers

Answer:

Step-by-step explanation:

Which equation represents a line parallel to the line whose equations is -2x + 3y =
-4 and passes through the point (1,3)?

Answers

Answer:

2.  y - 3 = 2/3 (x - 1).

Step-by-step explanation:

-2x + 3y = -4

3y = 2x - 4

y = 2/3 x - 4/3 - so the slope is  2/3.

The slope of a line parallel to it is also 2/3.

It also passes through the point (1, 3).

Using the point-slope form of  a line:

y - y1 = m(x - x1) where m = the slope and (x1, y1) is a point on the line, we have:

y - 3 = 2/3 (x - 1)  <--- is the required equation.

Answer:

2

Step-by-step explanation:

parallel lines have same slope with different intercept

y= mx+b

m is going to be same with different b

so the given function is:

-2x+3y=-43y= 2x-4y= 2/3x - 4/3

Given options:

1. y-3= - 2/3(x-1)  ⇒ y= -2/3x +3 + 2/3 ⇒ y= -2/3x +11/3

it has different slope, so is not parallel

2. y-3= 2/3(x-1) ⇒ y= 2/3x+3-2/3 ⇒ y= 2/3x + 7/3

it has same slope, so is parallelit should be passing through point (1,3)3= 2/3+7/3 ⇒ 3=3, yes it does

3. y-3= -2/3(x+1) ⇒ y= - 2/3x +3- 2/3 ⇒ y= -2/3x + 7/3

it has different slope, is not parallel

4. y-3= 2/3(x+1) ⇒ y= 2/3x +3+ 2/3 ⇒ y= 2/3x +11/3

it has same slope, so is parallelit should be passing through point (1,3)3= 2/3+11/3 ⇒ 3≠13/3, no it doesn't
Other Questions
What is a tense? Please explain French: Use depuis or il ya. 1. Nous nous connaisons....... trente ans. Please help. PLEASE HELP ME THIS IS URGENT I AM BEING TIMED!Melanie ordered books from a catalog. The prices of the books added up to $45.60. She had to pay a 6% tax and 8% for shipping. What was the total cost of her order? Don't forget to include the $ sign. 17 large and blue, 3 small and blue, 8 large and red, 12 small and red, how many large or blue Whats the proper way to say aint you got nobody? You are working as accounting information system(AIS) expert in H and H, a multinational entity(MNE) for couple of years inSpain. One of its subsidiaries in an emerging country of Asia is developing their AIS. Management selected you for this expatriate assignment for the next three years. Your boss will have a metting with you next week. What would you discuss with your boss? Find the area of the shape. Solve the given system of equations. This table shows the time it took a group of students to complete a quiz Target sells 24 bottles of water for $3.and 36 bottles of water for $4 Which is the better buy and by how much24 bottles for $3 by 1.50 per bottleB)24 bottles for 53 by 700 per bottle24 bottles for $3 by 75c per bottle36 bottles for $4 by 1 46 per bottle60 Help appreciated please and thank you Purchase-Related Transactions Using Perpetual Inventory System The following selected transactions were completed by Niles Co. during March of the current year: Mar. 1. Purchased merchandise from Haas Co., $13,900, terms FOB shipping point, 2/10, n/eom. Prepaid freight of $550 was added to the invoice. 5. Purchased merchandise from Whitman Co., $10,650, terms FOB destination, n/30. 10. Paid Haas Co. for invoice of March 1. 13. Purchased merchandise from Jost Co., $7,100, terms FOB destination, 1/10, n/30. 14. Issued debit memo to Jost Co. for $1,300 of merchandise returned from purchase on March 13. 18. Purchased merchandise from Fairhurst Company, $9,600, terms FOB shipping point, n/eom. 18. Paid freight of $310 on March 18 purchase from Fairhurst Company. 19. Purchased merchandise from Bickle Co., $13,800, terms FOB destination, 2/10, n/30. 23. Paid Jost Co. for invoice of March 13, less debit memo of March 14. 29. Paid Bickle Co. for invoice of March 19. 31. Paid Fairhurst Company for invoice of March 18. 31. Paid Whitman Co. for invoice of March 5. Required: Journalize the entries to record the transactions of Britt Co. for March. Mar. 1 Mar. 5 Mar. 10 Mar. 13 Mar. 14 Mar. 18-purchase Mar. 18-freight Mar. 19 Mar. 23 Mar. 29 Mar. 31-Fairhurst Mar. 31-Whitman Water flows from the bottom of a storage tank at a rate of r(t) = 400 8t liters per minute, where 0 t 50. Find the amount of water that flows from the tank during the first 30 minutes.______ Liters. The kitchen assistant is helping the chef to serve soup. The chef made 3 pots of soup and the assistant is putting 1/16 cups of cream on top of the soup in each bowl. There are 12 cups of cream. How many bowls of soup can the assistant help prepare? According to this passage, Emerson sees demand andsupply asWealth brings with it its own checks and balances. Thebasis of political economy is non interference. The onlysafe rule is found in the self-adjusting meter of demandand supply. Do not legislate.... Make equal laws: securelife and property, and you need not give alms. Open thedoors of opportunity to talent and virtue, and they will dothemselves justice, and property will not be in bad hands.In a free and just commonwealth, property rushes fromthe idle [lazy] and imbecile, to the industrious, brave, andpersevering-Culture, Behavior, Beauty,Ralph Waldo Emersona means of productionO a challenge to private ownershipO a principle that keeps the economy stableO a barrier to free trade A designer enlarged both the length and the width of a rectangular carpet by 60 percent. The new carpet was too large so the designer was asked to reduce its length and its width by 25 percent. By what percent was the area of the final item greater than the area of the original? What were the killing fields and the Cambodian genocide? Mention and explain the different types of checks, and is the type of the check effectthe indorsement? The equation y = - 8) is graphed in the zy-plane. Which of the following equations willhave a graph that is parallel to the graph of the above equation and have an X-intercept on thenegative x-axis? True or False: the Process of using electors is set up in the constitution.